LSAT and Law School Admissions Forum

Get expert LSAT preparation and law school admissions advice from PowerScore Test Preparation.

 Administrator
PowerScore Staff
  • PowerScore Staff
  • Posts: 8916
  • Joined: Feb 02, 2011
|
#84224
Complete Question Explanation

Assumption. The correct answer choice is (D)

The stimulus contains conditional reasoning, and can be diagrammed as follows:

     DS = democratic society exists
     SB = citizens establish strong bonds of mutual trust
     PCO = participation in civic organizations, political parties, and other groups outside the family

     Premises: DS :arrow: SB :arrow: PCO

     Conclusion: Widespread reliance on movies and electronic media for entertainment has an
                                  inherently corrosive effect on democracy.

Answer choice (A): This was the most commonly selected wrong answer, and this answer is
incorrect because it exaggerates the situation. The author does not assume that anyone would be
unable to form a strong bond of trust. The argument was clear about a corrosive effect on democracy.
This answer, if it were an assumption, would lead to the end of democracy. That result is too strong
for the author’s conclusion.

Answer choice (B): The author makes no assumption regarding organizations advancing their
agenda.

Answer choice (C): The argument is not about newspapers and print media.

Answer choice (D): This is the correct answer. This Supporter assumption connects the new element
in the conclusion back to the conditional relationship in the premises.

Answer choice (E): The author does not assume that closeness to the family is a bad thing, but that
one must also participate outside the family. This answer does not suggest otherwise, and it is not an
assumption of the argument.
 SherryZ
  • Posts: 124
  • Joined: Oct 06, 2013
|
#11955
Hi there! Thank you for your patience and time! You guys are the best! :lol:

June 2001 LSAT, Sec 3 LR, #5:

Fortunately, I picked the correct answer choice D. However, I was distracted by A.

Could you explain why A is wrong? I don't wanna be distracted again when I confront similar question next time.

Thank you very much!

Sincerely,
Sherry
 Jacques Lamothe
PowerScore Staff
  • PowerScore Staff
  • Posts: 50
  • Joined: Sep 24, 2013
|
#11964
Hi Sherry,

Answer choice (A) is attractive because assuming that answer is sufficient to prove the author's argument. If democracies really cannot exist without strong bonds of mutual trust and reliance on electronic media make forming those bonds impossible, then widespread reliance on electronic media will have a corrosive effect on democracy.

The problem with (A) is that it is too strong a claim to be a necessary assumption on which the argument depends. In this case, using the negation test can be a really helpful way to exclude answers like this more quickly. If those who rely on electronic media for entertainment are capable of forming strong bonds of mutual trust, is the author's argument invalid? Not in this case, because we can imagine reliance on electronic media having an effect on the formation of bonds, even if they do not make those bonds impossible.

In contrast, if relying on electronic media does not decrease the likelihood of participating in groups outside the family, that reliance would not impact the formation of bonds of trust since participation outside of the family is the only way in which those bonds form. This demonstrates that if you negate answer choice (D), the argument cannot be true. That makes D our correct answer.

I hope that help!

Jacques
 adlindsey
  • Posts: 90
  • Joined: Oct 02, 2016
|
#31928
I also had it between A & D, but went with D after applying the negation technique.
Answer choice (A) is attractive because assuming that answer is sufficient to prove the author's argument. If democracies really cannot exist without strong bonds of mutual trust and reliance on electronic media make forming those bonds impossible, then widespread reliance on electronic media will have a corrosive effect on democracy.

So this would make A the correct answer if it was a Justify question?
 Kristina Moen
PowerScore Staff
  • PowerScore Staff
  • Posts: 230
  • Joined: Nov 17, 2016
|
#31960
adlindsey,

Yes, you got it! That would mean that folks who relied on electronic media would not meet the requirements (strong bond) for a democratic society. However, answer choice (A) is not necessary.
 blade21cn
  • Posts: 100
  • Joined: May 21, 2019
|
#82786
The main difference between (A) and (D) is that (A) links up "reliance" and "NOT strong bond" directly, while (A) links up "reliance" and "NOT participation," which are next to each other on the logic chain. So can I eliminate (A) for skipping the middle guy, which is not necessary? But logically speaking, "reliance → NOT strong bond" and "reliance → NOT participation" are the same due to the premise that "NOT participation → NOT strong bond." Any thoughts? Thanks!
 Adam Tyson
PowerScore Staff
  • PowerScore Staff
  • Posts: 5153
  • Joined: Apr 14, 2011
|
#84251
That's not the real problem with answer A, blade21cn. The problem is that the answer is much too strong to be necessary in this argument. The author doesn't have to believe that anyone who relies on movies lacks that trust - they have to believe that widespread reliance on movies breaks down that trust. The author might be just fine with the idea that some people who rely on movies still manage to form those trusting bonds, so long as widespread reliance diminishes those bonds somewhat, thereby corroding democracy. It's about a cumulative effect, not an individual one.
 ltowns1
  • Posts: 61
  • Joined: May 16, 2017
|
#93963
Hi, just curious, had the answer said "anyone who watches movies and electronic media for entertainment are unable to form and strengthen bonds" would that make the answer correct?
 Adam Tyson
PowerScore Staff
  • PowerScore Staff
  • Posts: 5153
  • Joined: Apr 14, 2011
|
#94248
That would still be too strong an answer, ltowns1. The author doesn't have to assume that relying on movies makes it impossible to form those bonds. They just have to assume that doing so makes it harder to form them, or that it makes people less likely to form those bonds. It's that difference in the answers - "unable" vs "less likely" - that makes D the winner. Assumptions, like Must Be True answers, should not be so strong that they overstate the author's position.

Get the most out of your LSAT Prep Plus subscription.

Analyze and track your performance with our Testing and Analytics Package.